2021 AMC 12B Problems/Problem 8
- The following problem is from both the 2021 AMC 10B #14 and 2021 AMC 12B #8, so both problems redirect to this page.
Problem
Three equally spaced parallel lines intersect a circle, creating three chords of lengths
and
. What is the distance between two adjacent parallel lines?
Solution 1 (Pythagorean Theorem)
Since two parallel chords have the same length (
), they must be equidistant from the center of the circle. Let the perpendicular distance of each chord from the center of the circle be
. Thus, the distance from the center of the circle to the chord of length
is
and the distance between each of the chords is just
. Let the radius of the circle be
. Drawing radii to the points where the lines intersect the circle, we create two different right triangles:
- One with base
, height
, and hypotenuse
(
on the diagram)
- Another with base
, height
, and hypotenuse
(
on the diagram)
By the Pythagorean theorem, we can create the following system of equations:
Solving, we find
, so
.
-Solution by Joeya and diagram by Jamess2022(burntTacos).
Solution 2 (Coordinates)
Because we know that the equation of a circle is
where the center of the circle is
and the radius is
, we can find the equation of this circle by centering it on the origin. Doing this, we get that the equation is
. Now, we can set the distance between the chords as
so the distance from the chord with length 38 to the diameter is
.
Therefore, the following points are on the circle as the y-axis splits the chord in half, that is where we get our x value:
Now, we can plug one of the first two value in as well as the last one to get the following equations:
Subtracting these two equations, we get
- therefore, we get
. We want to find
because that's the distance between two chords. So, our answer is
.
~Tony_Li2007
Solution 3 (Stewart's Theorem)
If
is the requested distance, and
is the radius of the circle, Stewart's Theorem applied to
with cevian
gives
This simplifies to
. Similarly, another round of Stewart's Theorem applied to
with cevian
gives
This simplifies to
. Dividing the top equation by
and the bottom equation by
results in the system of equations
By transitive,
. Therefore
~Punxsutawney Phil
Video Solution by Hawk Math
https://www.youtube.com/watch?v=VzwxbsuSQ80
Video Solution by Punxsutawney Phil
Video Solution by OmegaLearn (Circular Geometry)
Video Solution by TheBeautyofMath
https://youtu.be/L1iW94Ue3eI?t=1118 (for AMC 10B)
https://youtu.be/kuZXQYHycdk?t=574 (for AMC 12B)
~IceMatrix
Video Solution by Interstigation
~Interstigation
See Also
| 2021 AMC 12B (Problems • Answer Key • Resources) | |
| Preceded by Problem 7 |
Followed by Problem 9 |
| 1 • 2 • 3 • 4 • 5 • 6 • 7 • 8 • 9 • 10 • 11 • 12 • 13 • 14 • 15 • 16 • 17 • 18 • 19 • 20 • 21 • 22 • 23 • 24 • 25 | |
| All AMC 12 Problems and Solutions | |
| 2021 AMC 10B (Problems • Answer Key • Resources) | ||
| Preceded by Problem 13 |
Followed by Problem 15 | |
| 1 • 2 • 3 • 4 • 5 • 6 • 7 • 8 • 9 • 10 • 11 • 12 • 13 • 14 • 15 • 16 • 17 • 18 • 19 • 20 • 21 • 22 • 23 • 24 • 25 | ||
| All AMC 10 Problems and Solutions | ||
These problems are copyrighted © by the Mathematical Association of America, as part of the American Mathematics Competitions. Error creating thumbnail: File missing